You are on page 1of 14

Industrial Organization: Markets and Strategies Paul Belleamme and Martin Peitz

published by Cambridge University Press

Part VI. Theory of competition policy Exercises Exercise 1 Industries with cartels Briey describe and analyze a case of your choice concerning a price- or quantity-xing cartel (please not OPEC). The following questions may be useful to bear in mind: What are the relevant characteristics of the industry? What was the scope of the cartel? How was the cartel enforced? What were the eects of the cartels? How did the competition authority or court argue and what was the decision, if any? [suggested length: 1-2 pages, timesnewroman 12 pt, doublespaced] Exercise 2 Collusion and pricing Two (advertising-free) newspapers compete in prices for an innite number of days. The monopoly prots (per day) in the newspaper market are M and the discount rate (per day) is . If the newspapers compete in prices, they both earn zero prots in the static Nash equilibrium. Finally, if the rms set the same price, they split the market equally and earn the same prots. 1. The newspapers would like to collude on the monopoly price. Write down the strategies that the newspapers could follow to achieve this outcome. Find the discount rates for which they are able to sustain the monopoly price using these strategies. 2. On Sundays, the newspapers sell a weekly magazine (that can be bought without buying the newspaper). The monopoly (competitive) prots when selling the magazine are also M (zero). 3. For which discount rates can the monopoly price be sustained only in the market for magazines? (Write down the equation that characterizes the solution.) Compare the solution found in question 1 and 2 and comment briey. 4. For which discount rates can the monopoly price be sustained both in the market for newspapers and in the market for magazines? (Write down the equation that characterizes the solution.)

Exercise 3 Collusion and pricing II Consider a homogeneous-product duopoly. The two rms in the market are assumed to have constant marginal costs of production equal to c. The two rms compete possibly over an innite time horizon. In each period they simultaneously set price pi , i = 1, 2. After each period the market is closed down with probability 1 . Market demand Q(p) is decreasing, where p = min{p1 , p2 }. Suppose, furthermore, that the monopoly problem is well dened, i.e. there is a solution pM = arg maxp pQ(p). If rms set the same price, they share total demand with weight for rm 1 and 1 for rm 2. Suppose that [1/2, 1). Suppose that rms use trigger strategies and Nash punishment. 1. Suppose that = 0. Derive the equilibrium of the game. 2. Suppose that > 0 and = 1/2. Derive the condition according to which rm 1 and rm 2 do not nd it protable to deviate from the collusive price pM . 3. Suppose that > 0 and > 1/2. Derive the condition according according to which pM is played along the equilibrium path. Show that the condition is the more stringent the higher . 4. Show that previous results in (3) also hold for any collusive price pC (c, pM ). 5. Suppose that > 0 and > 1/2 and that rm 1 can only adjust its price every periods. Derive the condition according to which pM is played along the equilibrium path. How does the time span inuence the condition?

Exercise 4 Parallel pricing and evidence of collusion A competition policy authority has noticed that the rms in the Lysine industry consistently charge very similar prices, and the suspicion is that they are colluding. Do you think that parallel pricing is proof of collusion? If not, what kind of evidence would you look for? Exercise 5 Collusion and quantity competition Consider the following market: Two rms compete in quantities, i.e., they are Cournot competitors. The rms produce at constant marginal costs equal to 20. The inverse demand curve in the market is given by P (q ) = 260 q .

1. Find the equilibrium quantities under Cournot competition as well as the quantity that a monopolist would produce. Calculate the equilibrium profits in Cournot duopoly and the monopoly prots. Suppose that the rms compete in this market for an innite number of periods. The discount factor (per period) is , (0, 1). 2. The rms would like to collude in order to restrict the total quantity produced to the monopoly quantity. Write down strategies that the rms could use to achieve this outcome. 3. For which values of is collusion sustainable using the strategies of question (b)? [Hint: Think carefully about what the optimal deviation is.]

Exercise 6 Cournot mergers: protability and welfare properties Consider a homogeneous good duopoly with linear demand P (q ) = 12 q , where q is the total industry output, and constant marginal costs c = 3. 1. Suppose that rms simultaneously set quantities. Determine the equilibrium (price, quantities, prot, welfare). 2. The rms consider to merge although their production costs are not affected. Determine the solution to this problem. Is such a merger profitable? What are the welfare eects of such a merger? 3. Suppose that the merger is eciency enhancing, leading to marginal costs cm < c. What are the welfare eects of such a merger. Do you possibly have to qualify your answer in (2)? 4. Consider the possibility of rm entry after the merger (the entrant produces at marginal costs c = 3 and has entry cost e). Suppose rst that the merger is not eciency-enhancing. Analyze such a market and comment on your result (depending on the entry cost e). Suppose next that the merger is eciency-enhancing, i.e. cm < 3. Depending on cm and e, when is a merger protable? [Hint: Calculate prots for cm = 1/2.] 5. Many countries scrutinize merger and sometimes block them (or impose remedies)? Discuss which factors should make the courts or the competition authority more inclined to block a merger.

Exercise 7 Cournot mergers and synergies. Consider a homogeneous-product Cournot oligopoly with 4 rms. Suppose that the inverse demand function is P (q ) = 64 q .

1. Suppose that rms incur a constant marginal cost c = 4. Characterize the Nash equilibrium of the game in which all rms simultaneously choose quantity. 2. Suppose that rms 1 and 2 consider to merge and that there are synergies leading to marginal costs cm < c. Characterize the Nash equilibrium. At which level cm (you may want to give an approximate number) are the two rms indierent whether to merge? 3. Is such a merger that just makes the two rms indierent between merging and non-merging consumer-welfare increasing? 4. At which level cm would the merger be consumer-welfare neutral? 5. Suppose that instead rms 1, 2, and 3 consider to merge. The new marginal cost of the merged rms is cn < c. At which level cn are the three rms indierent whether to merge? 6. Compare your ndings in (5) and (2). What can you say about incentives to merge in this case?

Exercise 8 Cournot mergers and demand Consider the following Cournot merger game. The inverse demand function is of the form P (q ) = a q where > 0 and there are n rms with constant marginal costs of production c. 1. Discuss the shape of the inverse demand function depending on . 2. Determine the Cournot equilibrium prots in this set-up. 3. Determine all n for which a single merger, i.e. going from n to n 1 rms, is protable.

Exercise 9 Burning ships Hernan Cortz, the Spanish conqueror (conquistador), is said to have burned his ships upon the arrival to Mexico. Why would he do such a thing? Exercise 10 Quantity commitment

Up to two rms are in a market in which quantities are the strategic variable. There are two periods; in the rst period rm 1 is a protected monopolist. In each of the two periods t = 1, 2 the inverse demand function P t is given by P t (xt ) = 20 xt . In each period the cost function of rm i is given by t t Ci (xi ) = 9 + 4xt i . Prots of a rm are the sum of its prots in each period (no discounting). Firms maximize prots by setting quantities. 1. Determine the monopoly solution. 2. Because of technological restrictions rm 1 has to choose the same quantity 2 1 in each period (x1 1 = x1 ). Observing x1 , rm 2 is considering to enter in 1 period 2. Determine the prot maximizing x2 2 given x1 . 3. Assume that rm 2 will enter in period 2. What quantity will rm 1 produce? Determine equilibrium prices, quantities, and prots. 4. Firm 2 only enters in period 2 if it can make positive prots. Determine the subgame perfect equilibrium of the two-period model.

Exercise 11 Strategic quantity choice Consider a market with two rms, A and B . The rms produce homogenous goods, compete in quantities, and face a constant marginal cost equal to 1/4. The timing is the following: First, rm A chooses its quantity qA . Then, after observing qA , rm B chooses its quantity qB . The price in the market is given by the inverse demand function P (q ) = 1 q , where q = qA + qB . 1. Find the subgame perfect Nash equilibrium. 2. Assume from now that there is an entry cost of e. Firm A is already established in the market, and rm B is considering whether to establish itself in the market or not. The timing is the following: First, rm A chooses its quantity qA . Then, after observing qA , rm B decides whether to enter the market and, in case of entry, how much to produce. 3. Write down rm B s prot function. 4. Assume for now that e = 1/10. Illustrate rm As prot as a function of qA when the reaction of rm B is taken into account. 5. Find the subgame perfect Nash equilibrium for all values of e > 0.

Exercise 12 Taxonomy of entry-related strategies I

Consider a market with dierentiated products. In the rst stage rm 1 is the incumbent rm and can invest an amount K1 0 in reducing its marginal costs, c(K1 ) = c K1 /10. In stage two rm 2 can decides about entering the market with constant marginal costs of c and entry costs of e. In stage three if entry takes place rms engage in price competition and face symmetric demand functions given by Di (pi , pj ) = A api + bpj (A > a > b > 0). If no entry takes place, rm 1 acts as a monopolist with demand, D1 (p1 ) = A ap1 . 1. Calculate the best response functions for both rms. Draw a graph. Argue graphically from now on: 2. Does an increase in K1 increase or decrease the prot of the entering rm? Does an increase in K1 make the incumbent tough or soft? 3. Does a marginal investment K1 increase or decrease the prot of the incumbent? (Assume that e is suciently low such that entry takes place for K1 close to zero. Moreover, assume A 10) 4. Use your answer of (2): Is entry deterrence via cost reduction possible in this setting? If your answer is YES, which numbers would you have to compare to decide whether entry deterrence is optimal? If your answer is NO, what do we have change in this model to induce entry deterrence? 5. Use your answer of (3): If entry accommodation is optimal how much should rm 1 invest in cost reduction? 6. How would you answer to (4) change if we consider a Cournot game instead? 7. How would you answer to (5) change if we consider a Cournot game instead?

Exercise 13 Taxonomy of entry-related strategies II Consider the market from the previous exercise again. 1. Use the best-response functions from the previous exercise to calculate equilibrium prices for c1 6= c2 in the Nash equilibrium in which rms set prises. 2. Use your result from (1) to show that p i = (A + ac)/(2a b), for i = 1, 2, if c1 = c2 = c. Now, use again that c1 (K1 ) = c (K1 /10) and c2 = c. Set c = 4, a = 2, b = 1, A = 10, and F = 7.95.

D 3. Show that the critical level of K1 to deter entry is below 0.5.

4. Suppose that investment in cost reduction is restricted to half units, i.e. K1 {0, 0.5, 1, 1.5, . . .}. Will rm 1 deter entry in a subgame perfect Nash equilibrium? State rm 1s optimal business strategy. 5. Reconsider your answer to (4) if rm 1 as a monopolist faces a demand of D1 (p1 ) = A ap1 + bp1 = A (a b)p1 . Exercise 14 Sequential quantity choice and entry Consider a market for a homogenous good with one incumbent rm (rm 1) and one potential entrant (rm 2). The interaction between the two rms evolves in two stages. In stage 1, rm 1 chooses its quantity q1 . In stage 2, after observing q1 , rm 2 decides whether or not to enter the market. If it enters, it incurs an entry cost e and chooses its own quantity, q2 . If rm 2 does not enter then q2 = 0 and rm 2 does not pay the entry cost e (rm 1 then is a monopoly). Assume that the inverse demand for the good is P = a (q1 + q2 ), and that the cost of production of each rm i is C (qi ) = qi /2. 1. Compute the range of e for which entry is blockaded. That is, compute rm 1s output when it operates as a monopolist, then given this quantity, compute the highest prot that rm 2 can earn if it decides to enter, and nally, compute the range of e for which entry is blockaded. 2. Now, suppose that e is suciently low to ensure that entry is not blockaded. Compute the quantities and prots of each rm when entry is accommodated. That is, compute the outputs that will be selected in a Stackelberg equilibrium and the resulting prots. (Instruction: rst, compute rm 2s best response function, br2 (q1 ). Second, substitute for br2 (q1 ) into rm 1s prot function and compute rm 1s prot-maximizing quantity q1 . Third, nd rm 2s best response against q1 , using rm 2s best response function. Finally, given the pair of quantities you found, compute the equilibrium prots). 3. Compute the lowest q1 for which entry is deterred. Compute rm 1s prots at this output level. 4. Given your answer in (3), show rm 2s best response function graphically in the quantities space (recall that rm 2 may wish to stay out of the market when q1 is relatively high). Show on the same graph the Stackelberg equilibrium you found in Section (3) and the lowest q1 for which entry is deterred. 5. Given your answers in (2) and (3), nd the range of e for which entry is accommodated, and the range of e for which it is deterred. Explain in no more than 3 sentences the intuition for the result (i.e., why is it natural to expect that entry is accommodated/deterred when e is relatively low/high). 7

Exercise 15 Capacity choice and entry Consider an industry for a homogenous product with a single rm (rm 1) that can produce at zero cost. The demand function in the industry is given by Q = a p. Now suppose that a second rm (rm 2) considers entry into the industry. Firm 2 can also produce at zero cost. If rm 2 enters, rms 1 and 2 compete by setting prices. Consumers buy from the rm that sets the lowest price. If both rms charge the same prices, consumers buy from rm 1. 1. Solve for the Nash equilibrium if rm 2 chooses to enter the industry. Would rm 2 wish to enter if entry required some initial investment? 2. Now suppose that before it enters, rm 2 can choose a capacity, x2 , and a price p2 (the capacity x2 means that rm 2 can produce no more than q2 = x2 units). Given q2 and p2 , rm 1 chooses its price and then consumers decide who to buy from. Compute the Nash equilibrium in the product market if rm 1 chooses to ght rm 2. What is rm 1s prot in this case? Show rm 1s prot in a graph that has quantity on the horizontal axis and price on the vertical axis. Would rm 2 choose to produce in that case? 3. Now suppose that rm 1 decides to accommodate the entry of rm 2. Compute the residual demand that rm 1 faces after rm 2 sells q2 units, and then write the maximization problem of rm 1 and solve it for p1 . What is rm 1s prot if it decides to accommodate rm 2s entry? Draw rm 1s prot in a graph that has quantity on the horizontal axis and price on the vertical axis. Would rm 2 wish to enter in this case? 4. Given your answers to (2) and (3), compute for each p2 the largest capacity that rm 2 can choose without inducing rm 1 to ght it. (Hint: to answer the question you need to solve a quadratic equation. The solution is given by the small root). 5. Show that the capacity you computed in (4) is decreasing with p2 . Explain the intuition for your answer. Given your answer, explain how rm 2 will choose its price. Computing p2 is too complicated; you are just asked to explain in words how rm 2 chooses p2 .)

Exercise 16 Investment and incumbency Consider a dierentiated product market. At the rst stage rm 1 is the incumbent rm and can invest an amount I1 0 in reducing its marginal costs, c(I1 ) = c I1 /10. At stage two rm 2 decides whether to enter the market with constant marginal costs of c and entry costs of e, which is sunk at this stage. At stage three, if entry has taken place, rms engage in quantity competition and face inverse demand functions given by Pi (qi , qj ) = a bqi dqj (a > b > d > 0). If no entry takes place, rm 1 acts as a monopolist with inverse demand, P m (q1 ) = a bq1 . 8

1. Calculate the best response functions for both rms. Draw a graph. 2. Does an increase in I1 increase or decrease the prot of the entering rm? Does an increase in I1 make the incumbent tough or soft? 3. Does a marginal investment I1 > 0 increase or decrease the prot of the incumbent? (Assume that e is suciently low such that entry takes place for I1 close to zero.) 4. If entry accommodation is optimal how much should rm 1 invest in cost reduction? (Use your answer of (3).) 5. Is entry deterrence via cost reduction possible and protable in this setting? Discuss your results in the light of the taxonomy developed in the book.

Exercise 17 Competition and entry Consider a homogeneous good duopoly with linear demand P (q ) = 1 q , where q is the total industry output. Suppose that rms are quantity setters and rms incur constant marginal costs of production ci . 1. Suppose that rms have constant marginal costs of production c. Determine the Nash equilibrium in quantities (report prices, quantities, prot, welfare) 2. Reconsider your answer in (1) because of the following: A tabloid runs a series on consumers paying excessive prices. The government considers introducing a non-negative special sales tax t 0 per unit on this product (and plans to use the revenues for some project from which nobody benets). Determine the welfare-maximizing tax rate (the government is assumed to be able to commit to the tax; welfare is total surplus which includes tax revenues). Discuss your result. What would be your conclusion if the government was considering subsidizing the rm? 3. Return to the case without taxes. Consider now the duopoly with c1 = 0 and c2 = c [0, 1]. Determine the equilibrium (price, quantities, prot, welfare). 4. Consider now an extended model in which only rm 1 is necessarily present. At stage 1, rm 1 can make an investment I after which rm 2s marginal costs is c2 = 1/2 instead of c2 = 0. Afterwards, rm 2 observes the investment decision of rm 1 and, at stage 2, decides whether to enter at a negligible entry cost e > 0. At stage 3, active rms set quantities simultaneously. Determine the subgame perfect equilibrium of this game. Discuss your result in the light of what you have learnt reading about entry-related strategies (max 3 sentences). 9

5. Consider now a dierent entry model. Both rms have zero marginal costs of production but consumers have become accustomed to product 1 (even if they did not consume it themselves). Therefore, consumers are willing to pay 1/2 money units less for product 2 than for product 1. The inverse demand function P (q ) = 1 q gives demand for product 1. At stage 1, the potential entrant, rm 2, considers to enter the market at an entry cost e > 0. (There is no investment stage in this game.) At stage 2, rms set quantities simultaneously. Report the prot function for each rm. Determine the equilibrium in case rm 2 has entered (report prices, quantities, prot, welfare). Determine the subgame perfect equilibrium and comment on your result. You may want to reuse some of the results derived above.

Exercise 18 Non-linear pricing in the supply chain A monopolist produces a good with constant marginal cost equal to c, c < 1. Assume for now that all consumers have the demand Q(p) = 1 p. The population is of size 1. 1. Suppose that the monopolist cannot discriminate in any way among the consumers and has to charge a uniform price, pU . Calculate both the price that maximizes prots and the prots that correspond to this price. 2. Suppose now that the monopolist can charge a two-part tari (m, p) where m is the xed fee and p is the price per unit. Expenditure then is m + pq . Calculate the two-part tari that maximizes prots and the prots that correspond to this tari. Compare pU and p and comment briey. 3. Compare the situation with a uniform price and a two-part tari in terms of welfare (a verbal argument is sucient). 4. Assume now instead that there are two types of consumers. The consumers of type 1 have the demand Q1 (p) = 1 p, and the consumers of type 2 have the demand Q2 (p) = 1 p/2. The population is of size 1 and there are equally many consumers of the two types. Finally, it is assumed in this question that c = 1/2. Calculate the two-part tari that maximizes the prots of the monopolist. Compare the two-part taris found in questions (2) and (3) for c = 1/2 and comment briey.

Exercise 19 RPM RPM was common in a number of industries. In particular, it could be observed in the clothing, consumer electronics, and food industry. What is the probable motivation for rms to use RPM in these industries. Discuss the likely welfare consequences in these industries.

10

Exercise 20 Vertical contracting A buyer wants to buy one unit of a good from an incumbent seller. The buyers valuation of the good is 1, while the sellers cost of producing it is 1/2. Before the parties trade, a rival seller enters the market and his cost, c, is distributed on the unit interval according to a distribution function with density g (c). The two sellers then simultaneously make price oers and the buyer trades with the seller who oers the lowest price. If the two sellers oer the same price the buyer buys from the seller whose cost is lower. 1. Determine the price that the buyer pays in equilibrium, p , as a function of c. Given p , write the payos of the expected payos of the buyer and the two sellers. 2. Suppose that the distribution of c is uniform. Show p and the expected payos of the parties graphically (put c on the horizontal axis and the equilibrium price function on the vertical axis and show the payos by pointing out the appropriate areas in the graph). 3. Now suppose that the incumbent seller oers the buyer a contract before the entrant shows up. The contract requires the buyer to pay the incumbent seller the amount m regardless of whether he buys from him or from the entrant, and gives the buyer an option to buy from the incumbent at a price of p (this is equivalent to giving the buyer an option to buy at a price m + p and requiring him to pay liquidated damages of m if he switches to the entrant). If the buyer rejects the contract things are as in part (1). Given p and c, what is the price that the buyer will end up paying for the good? Using your answer, write the expected payos of the buyer and the two sellers as a function of p and m. 4. Explain why the incumbent seller will choose p by maximizing the sum of his expected payo, I , and the buyers expected payos, UB . 5. Write the rst-order condition for p and show that the prot-maximizing price of the incumbent seller, p , is such that p < 1/2. Also show that if g (0) > 0 then p > 0. 6. Explain why the contract is socially inecient. Is the outcome in part (1) socially ecient? Explain the intuition for your answer. 7. Compute p assuming that the distribution of c is uniform, and show the expected payos of the parties and the social loss graphically (again, put c on the horizontal axis and the equilibrium price function on the vertical axis). 8. Compute p under the assumption that G(c) = c , where > 0. How does p vary with ? Give an intuition for this result.

11

Exercise 21 Franchising A monopolistic manufacturer produces a good that is sold to three retailers. The manufacturer has constant marginal cost equal to c < 1 2 . The retailers are monopolists in three dierent cities. The marginal cost of the retailers is equal to the wholesale price of the good. Each of the retailers faces a demand function Qb = 1 b p where p is the retail and b is a variable characterizing the local demand function. Each retailer knows the value of b in its city. Furthermore, it is common knowledge that b = 1 in one city and b = z in two cities, z (1, 2]. The manufacturer oers each of the retailers a two-part tari consisting of a wholesale price w and a franchise fee f . Let us assume that the retailers accept any contract that results in nonnegative prots. The retailers choose the retail price p in their market (i.e., there is no resale price maintenance). 1. Let w < 1 2 and take f as given. Find the price that a retailer sets as a function of b. Who earns the highest prot, retailers with b = 1 or b = z ? 2. Assume in the rest of the exercise that z = 2. Suppose rst that the manufacturer knows the value of b in all three cities and that z = 2. What contract will the manufacturer oer to the retailers? Is the franchise fee the same for all retailers? Is it possible for the retailer to extract all prots in the vertical chain? Assume from now on that the manufacturer does not know the retailers individual b. However, the manufacturer knows that two of the retailers have b = z = 2 and that one retailer has b = 1. Assume also that the manufacturer wishes to serve all retailers. 3. Find the optimal franchise fee as a function of the wholesale price w. 4. Find the optimal wholesale price as a function of c. Is the wholesale price greater or less than c? 5. Can the manufacturer extract all prots by setting w and f optimally? 6. Assume now instead that c = 1/4. Show that it is optimal for the manufacturer only to sell to the retailer with b = 1. What happens if c 0? Exercise 22 Exclusive dealing Suppose that two rms produce at constant marginal costs c. There are two periods and m buyers. Each buyer has an inverse demand curve: P (qI + qE ) = 1 (qI + qE ) where qI is the quantity sold by the incumbent and qE is the quantity sold by the entrant. In the rst period, there is only the incumbent in the market. Thus, the incumbent produces the monopoly quantity. The incumbent has marginal cost equal to cI . In the second period, an entrant with constant marginal cost equal to zero enters into the market. The entry is foreseen by the buyers. After entry, the two rms compete la Cournot. In the rst period, the incumbent oers the buyers a fee for an exclusive dealing agreement (take-it-or-leave-it). If a buyer accepts the oer, she cannot buy from the entrant in the second period. 12

1. Suppose that the incumbent and entrant are equally ecient, i.e. cI = 0. What is the maximal fee for an exclusive dealing agreement that the incumbent is willing to oer to a buyer? What is the minimum fee that a buyer is willing to accept for signing an exclusive dealing agreement? Will there be exclusive dealing in equilibrium? 2. Consider a general marginal cost of the incumbent cI . For which values of cI will exclusive dealing arise in equilibrium?

Exercise 23 Long-term contracts, upgrades, and exclusion Consider a market for a base good that is sold over two periods. In period 2, also an upgrade may become available. For simplicity, set the mass of consumers equal to 1 and marginal costs equal to zero. Firm 1 can be active in periods 1 and 2, rm 2 can only be active in period 2. At the beginning of period 2, rms decide simultaneously whether to upgrade. Suppose that rms maximize the sum of prots in periods 1 and 2. The willingness-to-pay without upgrades is V per consumer in each period. An upgrade by rm 1 leads to a surplus of r + 1 , while an upgrade by rm 2 would lead to a surplus of r + 2 . Firm 2 is assumed to be more ecient, 2 > 1 . The upgrading cost is C . Suppose furthermore that 1 > C . This assumption means that upgrading is socially superior to not upgrading even if it is done by the less ecient rm. 1. Characterize the equilibrium if rms can only oer short-term contracts, i.e., rm 1, when selling to consumers in period 1, cannot make them sign a contract that binds consumers to buy from it in period 2. 2. Characterize the equilibrium if rm 1 can oer a long-term contract that does not allow consumers or prevents them from buying from rm 2. Discuss your result.

Exercise 24 Vertical duopoly and vertical integration. Consider a vertical duopoly with exclusive dealing contracts in place, i.e., upstream rm i only sells to downstream rm , i = 1, 2. Suppose that, at stage 1, upstream rms and then, at stage 2, downstream rms set prices. Downstream demand is of the form Qi (pi , pj ) = 1 bpi + dpj . Upstream rms have zero marginal costs of production and set their wholesale price. Consider subgame perfect equilibria. 1. Characterize equilibrium upstream and downstream prices. 2. Suppose that b = d = 1. Characterize the equilibrium if rms indexed by 1 have vertically integrated (so that the integrated rms transfer price is 0). 13

3. Are there incentives for vertical integration (for b = d = 1)? Discuss your results.

14

You might also like